Stay ahead of learning milestones! Enroll in a class over the summer!

G
Topic
First Poster
Last Poster
k a April Highlights and 2025 AoPS Online Class Information
jlacosta   0
Wednesday at 3:18 PM
Spring is in full swing and summer is right around the corner, what are your plans? At AoPS Online our schedule has new classes starting now through July, so be sure to keep your skills sharp and be prepared for the Fall school year! Check out the schedule of upcoming classes below.

WOOT early bird pricing is in effect, don’t miss out! If you took MathWOOT Level 2 last year, no worries, it is all new problems this year! Our Worldwide Online Olympiad Training program is for high school level competitors. AoPS designed these courses to help our top students get the deep focus they need to succeed in their specific competition goals. Check out the details at this link for all our WOOT programs in math, computer science, chemistry, and physics.

Looking for summer camps in math and language arts? Be sure to check out the video-based summer camps offered at the Virtual Campus that are 2- to 4-weeks in duration. There are middle and high school competition math camps as well as Math Beasts camps that review key topics coupled with fun explorations covering areas such as graph theory (Math Beasts Camp 6), cryptography (Math Beasts Camp 7-8), and topology (Math Beasts Camp 8-9)!

Be sure to mark your calendars for the following events:
[list][*]April 3rd (Webinar), 4pm PT/7:00pm ET, Learning with AoPS: Perspectives from a Parent, Math Camp Instructor, and University Professor
[*]April 8th (Math Jam), 4:30pm PT/7:30pm ET, 2025 MATHCOUNTS State Discussion
April 9th (Webinar), 4:00pm PT/7:00pm ET, Learn about Video-based Summer Camps at the Virtual Campus
[*]April 10th (Math Jam), 4:30pm PT/7:30pm ET, 2025 MathILy and MathILy-Er Math Jam: Multibackwards Numbers
[*]April 22nd (Webinar), 4:00pm PT/7:00pm ET, Competitive Programming at AoPS (USACO).[/list]
Our full course list for upcoming classes is below:
All classes run 7:30pm-8:45pm ET/4:30pm - 5:45pm PT unless otherwise noted.

Introductory: Grades 5-10

Prealgebra 1 Self-Paced

Prealgebra 1
Sunday, Apr 13 - Aug 10
Tuesday, May 13 - Aug 26
Thursday, May 29 - Sep 11
Sunday, Jun 15 - Oct 12
Monday, Jun 30 - Oct 20
Wednesday, Jul 16 - Oct 29

Prealgebra 2 Self-Paced

Prealgebra 2
Sunday, Apr 13 - Aug 10
Wednesday, May 7 - Aug 20
Monday, Jun 2 - Sep 22
Sunday, Jun 29 - Oct 26
Friday, Jul 25 - Nov 21

Introduction to Algebra A Self-Paced

Introduction to Algebra A
Monday, Apr 7 - Jul 28
Sunday, May 11 - Sep 14 (1:00 - 2:30 pm ET/10:00 - 11:30 am PT)
Wednesday, May 14 - Aug 27
Friday, May 30 - Sep 26
Monday, Jun 2 - Sep 22
Sunday, Jun 15 - Oct 12
Thursday, Jun 26 - Oct 9
Tuesday, Jul 15 - Oct 28

Introduction to Counting & Probability Self-Paced

Introduction to Counting & Probability
Wednesday, Apr 16 - Jul 2
Thursday, May 15 - Jul 31
Sunday, Jun 1 - Aug 24
Thursday, Jun 12 - Aug 28
Wednesday, Jul 9 - Sep 24
Sunday, Jul 27 - Oct 19

Introduction to Number Theory
Thursday, Apr 17 - Jul 3
Friday, May 9 - Aug 1
Wednesday, May 21 - Aug 6
Monday, Jun 9 - Aug 25
Sunday, Jun 15 - Sep 14
Tuesday, Jul 15 - Sep 30

Introduction to Algebra B Self-Paced

Introduction to Algebra B
Wednesday, Apr 16 - Jul 30
Tuesday, May 6 - Aug 19
Wednesday, Jun 4 - Sep 17
Sunday, Jun 22 - Oct 19
Friday, Jul 18 - Nov 14

Introduction to Geometry
Wednesday, Apr 23 - Oct 1
Sunday, May 11 - Nov 9
Tuesday, May 20 - Oct 28
Monday, Jun 16 - Dec 8
Friday, Jun 20 - Jan 9
Sunday, Jun 29 - Jan 11
Monday, Jul 14 - Jan 19

Intermediate: Grades 8-12

Intermediate Algebra
Monday, Apr 21 - Oct 13
Sunday, Jun 1 - Nov 23
Tuesday, Jun 10 - Nov 18
Wednesday, Jun 25 - Dec 10
Sunday, Jul 13 - Jan 18
Thursday, Jul 24 - Jan 22

Intermediate Counting & Probability
Wednesday, May 21 - Sep 17
Sunday, Jun 22 - Nov 2

Intermediate Number Theory
Friday, Apr 11 - Jun 27
Sunday, Jun 1 - Aug 24
Wednesday, Jun 18 - Sep 3

Precalculus
Wednesday, Apr 9 - Sep 3
Friday, May 16 - Oct 24
Sunday, Jun 1 - Nov 9
Monday, Jun 30 - Dec 8

Advanced: Grades 9-12

Olympiad Geometry
Tuesday, Jun 10 - Aug 26

Calculus
Tuesday, May 27 - Nov 11
Wednesday, Jun 25 - Dec 17

Group Theory
Thursday, Jun 12 - Sep 11

Contest Preparation: Grades 6-12

MATHCOUNTS/AMC 8 Basics
Wednesday, Apr 16 - Jul 2
Friday, May 23 - Aug 15
Monday, Jun 2 - Aug 18
Thursday, Jun 12 - Aug 28
Sunday, Jun 22 - Sep 21
Tues & Thurs, Jul 8 - Aug 14 (meets twice a week!)

MATHCOUNTS/AMC 8 Advanced
Friday, Apr 11 - Jun 27
Sunday, May 11 - Aug 10
Tuesday, May 27 - Aug 12
Wednesday, Jun 11 - Aug 27
Sunday, Jun 22 - Sep 21
Tues & Thurs, Jul 8 - Aug 14 (meets twice a week!)

AMC 10 Problem Series
Friday, May 9 - Aug 1
Sunday, Jun 1 - Aug 24
Thursday, Jun 12 - Aug 28
Tuesday, Jun 17 - Sep 2
Sunday, Jun 22 - Sep 21 (1:00 - 2:30 pm ET/10:00 - 11:30 am PT)
Monday, Jun 23 - Sep 15
Tues & Thurs, Jul 8 - Aug 14 (meets twice a week!)

AMC 10 Final Fives
Sunday, May 11 - Jun 8
Tuesday, May 27 - Jun 17
Monday, Jun 30 - Jul 21

AMC 12 Problem Series
Tuesday, May 27 - Aug 12
Thursday, Jun 12 - Aug 28
Sunday, Jun 22 - Sep 21
Wednesday, Aug 6 - Oct 22

AMC 12 Final Fives
Sunday, May 18 - Jun 15

F=ma Problem Series
Wednesday, Jun 11 - Aug 27

WOOT Programs
Visit the pages linked for full schedule details for each of these programs!


MathWOOT Level 1
MathWOOT Level 2
ChemWOOT
CodeWOOT
PhysicsWOOT

Programming

Introduction to Programming with Python
Thursday, May 22 - Aug 7
Sunday, Jun 15 - Sep 14 (1:00 - 2:30 pm ET/10:00 - 11:30 am PT)
Tuesday, Jun 17 - Sep 2
Monday, Jun 30 - Sep 22

Intermediate Programming with Python
Sunday, Jun 1 - Aug 24
Monday, Jun 30 - Sep 22

USACO Bronze Problem Series
Tuesday, May 13 - Jul 29
Sunday, Jun 22 - Sep 1

Physics

Introduction to Physics
Wednesday, May 21 - Aug 6
Sunday, Jun 15 - Sep 14
Monday, Jun 23 - Sep 15

Physics 1: Mechanics
Thursday, May 22 - Oct 30
Monday, Jun 23 - Dec 15

Relativity
Sat & Sun, Apr 26 - Apr 27 (4:00 - 7:00 pm ET/1:00 - 4:00pm PT)
Mon, Tue, Wed & Thurs, Jun 23 - Jun 26 (meets every day of the week!)
0 replies
jlacosta
Wednesday at 3:18 PM
0 replies
Wot n' Minimization
y-is-the-best-_   24
N 6 minutes ago by maromex
Source: IMO SL 2019 A3
Let $n \geqslant 3$ be a positive integer and let $\left(a_{1}, a_{2}, \ldots, a_{n}\right)$ be a strictly increasing sequence of $n$ positive real numbers with sum equal to 2. Let $X$ be a subset of $\{1,2, \ldots, n\}$ such that the value of
\[
\left|1-\sum_{i \in X} a_{i}\right|
\]is minimised. Prove that there exists a strictly increasing sequence of $n$ positive real numbers $\left(b_{1}, b_{2}, \ldots, b_{n}\right)$ with sum equal to 2 such that
\[
\sum_{i \in X} b_{i}=1.
\]
24 replies
y-is-the-best-_
Sep 23, 2020
maromex
6 minutes ago
Functional equations
hanzo.ei   12
N 15 minutes ago by truongphatt2668
Source: Greekldiot
Find all $f: \mathbb R_+ \rightarrow \mathbb R_+$ such that $f(xf(y)+f(x))=yf(x+yf(x)) \: \forall \: x,y \in \mathbb R_+$
12 replies
hanzo.ei
Mar 29, 2025
truongphatt2668
15 minutes ago
Kosovo Mathematical Olympiad 2016 TST , Problem 1
dangerousliri   2
N 28 minutes ago by navier3072
Solve equation in real numbers

$\sqrt{x+\sqrt{4x+\sqrt{16x+\sqrt{…+\sqrt{4^nx+3}}}}}-\sqrt{x}=1$
2 replies
dangerousliri
Jan 9, 2017
navier3072
28 minutes ago
An almost identity polynomial
nAalniaOMliO   4
N 44 minutes ago by EmersonSoriano
Source: Belarusian National Olympiad 2025
Let $n$ be a positive integer and $P(x)$ be a polynomial with integer coefficients such that $P(1)=1,P(2)=2,\ldots,P(n)=n$.
Prove that $P(0)$ is divisible by $2 \cdot 3 \cdot \ldots \cdot n$.
4 replies
1 viewing
nAalniaOMliO
Mar 28, 2025
EmersonSoriano
44 minutes ago
Definite integral
PolyaPal   3
N Yesterday at 7:45 PM by PolyaPal
If $n$ is a nonnegative integer, evaluate $\int_0^1 \frac{x^n}{1 + x^2}\,dx$.
3 replies
PolyaPal
Mar 28, 2025
PolyaPal
Yesterday at 7:45 PM
Chebyshev polynomial and prime number
mofidy   0
Yesterday at 5:51 PM
Let $U_n(x)$ be a Chebyshev polynomial of the second kind. If n>2 and x > 2 is a integer, Could $U_n(x) -1$ be a prime number?
Thanks.
0 replies
mofidy
Yesterday at 5:51 PM
0 replies
Spheres and a point source of light
mofidy   4
N Yesterday at 5:40 PM by mofidy
How many spheres are needed to shield a point source of light?
Unfortunately, I didn't find a suitable solution for it on the page below:
https://artofproblemsolving.com/community/c4h1469498p8521602
Here too, two different solutions are given:
https://math.stackexchange.com/questions/2791186
4 replies
mofidy
Mar 11, 2025
mofidy
Yesterday at 5:40 PM
determinant of the matrix with power series element
jokerjoestar   0
Yesterday at 4:31 PM
Given the function

\[
f_k(x) = 1 + 2x + 3x^2 + \dots + (k+1)x^k,
\]
show that

\[
\begin{vmatrix} 
f_0(1) & f_1(1) & f_2(1) & \dots & f_{2023}(1) \\ 
f_0(2) & f_1(2) & f_2(2) & \dots & f_{2023}(2) \\ 
\vdots & \vdots & \vdots & \ddots & \vdots \\ 
f_0(2024) & f_1(2024) & f_2(2024) & \dots & f_{2023}(2024) 
\end{vmatrix}
= \prod_{k=1}^{2024} k!.
\]
0 replies
jokerjoestar
Yesterday at 4:31 PM
0 replies
Matrix problem
hef4875   3
N Yesterday at 3:43 PM by loup blanc
The matrix \( A = (a_{ij}) \in Mat_p(\mathbb{C}) \) is defined by the conditions
\( a_{12} = a_{23} = \dots = a_{(p-1)p} = 1 \) and \( a_{ij} = 0 \) for a set of indices \( (i,j) \).
Prove that there do not exist nonzero matrices \( B, C \in Mat_p(\mathbb{C}) \) satisfying the equation
\[
(I_p + A)^n = B^n + C^n.
\]$\forall$ $n$ is a postive integer.
3 replies
hef4875
Mar 26, 2025
loup blanc
Yesterday at 3:43 PM
A hard inequality
Butterfly   1
N Yesterday at 2:50 PM by jestrada

let $x_1,x_2,\cdots$ be all of the extrem points of $f(x)=x\cos\left(\frac{1}{x}\right)(x>0)$. Prove $f^2(x_1)+f^2(x_2)+\cdots \le \frac{\pi}{6{\rm e}}.$
1 reply
Butterfly
Yesterday at 9:43 AM
jestrada
Yesterday at 2:50 PM
Easy matrix equation involving invertibility
Ciobi_   2
N Yesterday at 12:59 PM by Moubinool
Source: Romania NMO 2025 11.2
Let $n$ be a positive integer, and $a,b$ be two complex numbers such that $a \neq 1$ and $b^k \neq 1$, for any $k \in \{1,2,\dots ,n\}$. The matrices $A,B \in \mathcal{M}_n(\mathbb{C})$ satisfy the relation $BA=a I_n + bAB$. Prove that $A$ and $B$ are invertible.
2 replies
Ciobi_
Wednesday at 1:46 PM
Moubinool
Yesterday at 12:59 PM
Strange limit
Snoop76   6
N Yesterday at 12:37 PM by Figaro
Find: $\lim_{n \to \infty} n\cdot\sum_{k=1}^n \frac 1 {k(n-k)!}$
6 replies
Snoop76
Mar 29, 2025
Figaro
Yesterday at 12:37 PM
Binomial inequality
Snoop76   10
N Yesterday at 11:51 AM by Snoop76
Is it true? $$\sum_{k=0}^n (2k-1)!!{n\choose k} >\left(\frac{2n}{e}\right)^n\sqrt{2e}$$
10 replies
Snoop76
Feb 2, 2025
Snoop76
Yesterday at 11:51 AM
An interesting limit
Alphaamss   0
Yesterday at 9:16 AM
Suppose $$x_1=\frac\pi2,\quad x_{n+1}=x_n-\frac{\sin x_n}{n+1},$$I can prove that the sequence $\{nx_n\}$ is convergent by monotone bounded convergence theorem.
Is there any method to compute the limit of $\{nx_n\}$, or give the asymptotic representation of $\{nx_n\}$? Any help and hints will welcome!
0 replies
Alphaamss
Yesterday at 9:16 AM
0 replies
2021china tst pure geo3
mathematics2003   24
N Jan 10, 2025 by HamstPan38825
Source: 2021ChinaTST test4 day1 P2
Let triangle$ABC(AB<AC)$ with incenter $I$ circumscribed in $\odot O$. Let $M,N$ be midpoint of arc $\widehat{BAC}$ and $\widehat{BC}$, respectively. $D$ lies on $\odot O$ so that $AD//BC$, and $E$ is tangency point of $A$-excircle of $\bigtriangleup ABC$. Point $F$ is in $\bigtriangleup ABC$ so that $FI//BC$ and $\angle BAF=\angle EAC$. Extend $NF$ to meet $\odot O$ at $G$, and extend $AG$ to meet line $IF$ at L. Let line $AF$ and $DI$ meet at $K$. Proof that $ML\bot NK$.
24 replies
mathematics2003
Apr 13, 2021
HamstPan38825
Jan 10, 2025
2021china tst pure geo3
G H J
G H BBookmark kLocked kLocked NReply
Source: 2021ChinaTST test4 day1 P2
The post below has been deleted. Click to close.
This post has been deleted. Click here to see post.
mathematics2003
16 posts
#1 • 2 Y
Y by Rounak_iitr, ys-lg
Let triangle$ABC(AB<AC)$ with incenter $I$ circumscribed in $\odot O$. Let $M,N$ be midpoint of arc $\widehat{BAC}$ and $\widehat{BC}$, respectively. $D$ lies on $\odot O$ so that $AD//BC$, and $E$ is tangency point of $A$-excircle of $\bigtriangleup ABC$. Point $F$ is in $\bigtriangleup ABC$ so that $FI//BC$ and $\angle BAF=\angle EAC$. Extend $NF$ to meet $\odot O$ at $G$, and extend $AG$ to meet line $IF$ at L. Let line $AF$ and $DI$ meet at $K$. Proof that $ML\bot NK$.
Z K Y
The post below has been deleted. Click to close.
This post has been deleted. Click here to see post.
MP8148
888 posts
#2 • 8 Y
Y by enzoP14, Modesti, GeoKing, Mango247, Mango247, Om245, Rounak_iitr, ys-lg
[asy]
size(8cm);
defaultpen(fontsize(10pt));

pair A = dir(125), B = dir(210), C = dir(330), M = dir(90), N = dir(270), D = dir(55), I = incenter(A,B,C), T = intersectionpoint(unitcircle,I--I+dir(M--I)*100), F = extension(A,T,I,I+dir(0)), G = F+dir(N--F)*abs(A-F)*abs(T-F)/abs(N-F), L = extension(A,G,I,F), K = extension(D,I,A,T), S = extension(M,L,N,K), T1 = 2*foot(T,M,N)-T;

draw(A--B--C--A--N^^unitcircle, heavyblue);
draw(D--A^^L--I^^B--C^^T--T1, heavyblue+linewidth(1.2));
draw(A--T--M^^D--K, orange);
draw(N--G^^A--L, purple);
draw(M--S^^N--S, red);
draw(L--S, red+dashed);
draw(circumcircle(A,G,F), cyan);
draw(circumcircle(M,F,I), magenta);
draw(S--T1, heavygreen+dashed);

dot("$A$", A, dir(120));
dot("$B$", B, dir(210));
dot("$C$", C, dir(330));
dot("$D$", D, dir(60));
dot("$M$", M, dir(90));
dot("$N$", N, dir(270));
dot("$I$", I, dir(330));
dot("$F$", F, dir(45));
dot("$K$", K, dir(210));
dot("$G$", G, dir(135));
dot("$S$", S, dir(150));
dot("$L$", L, dir(210));
dot("$T$", T, dir(225));
dot("$T'$", T1, dir(315));
[/asy]
Let $T = \overline{MI} \cap \overline{AF}$ be the $A$-mixtilinear intouch point, and let $S = \overline{NK} \cap (ABC)$. Since $\angle NSM = 90^\circ$, we just want $\overline{AG}$, $\overline{MS}$, $\overline{IF}$ concur. By (degenerate) Reim's we have $AGFI$ cyclic, so by radical axis it suffices to show $MIFS$ cyclic.

Let $T' = \overline{SF} \cap (ABC)$. Then $$(AN;TT') \overset{S}= (AK;TF) \overset{I}= (AD;\overline{TM} \cap \overline{AD}, \infty_{AD}) = \frac{TA}{TD},$$which implies $\overline{TT'} \parallel \overline{BC}$, from where it follows by Reim's that $MIFS$ is cyclic, as desired.

@2below it's because $(AN;TT')$ only depends on $T'$, and when $\overline{TT'} \parallel \overline{BC}$ the equation is true. Technically the harmonic conjugate of $T'$ would also work, but I was too lazy to address that (should be clear from $G-F-N$ that $T'$ must lie on the opposite side of $\overline{AN}$ as $T$).
This post has been edited 3 times. Last edited by MP8148, Apr 21, 2021, 6:10 PM
Z K Y
The post below has been deleted. Click to close.
This post has been deleted. Click here to see post.
W.R.O.N.G
41 posts
#3 • 1 Y
Y by Rounak_iitr
Solution?
This post has been edited 2 times. Last edited by W.R.O.N.G, Apr 14, 2021, 9:13 AM
Reason: added an important part of the proof
Z K Y
The post below has been deleted. Click to close.
This post has been deleted. Click here to see post.
hydo2332
435 posts
#4 • 1 Y
Y by Rounak_iitr
MP8148 wrote:
[asy]
size(8cm);
defaultpen(fontsize(10pt));

pair A = dir(125), B = dir(210), C = dir(330), M = dir(90), N = dir(270), D = dir(55), I = incenter(A,B,C), T = intersectionpoint(unitcircle,I--I+dir(M--I)*100), F = extension(A,T,I,I+dir(0)), G = F+dir(N--F)*abs(A-F)*abs(T-F)/abs(N-F), L = extension(A,G,I,F), K = extension(D,I,A,T), S = extension(M,L,N,K), T1 = 2*foot(T,M,N)-T;

draw(A--B--C--A--N^^unitcircle, heavyblue);
draw(D--A^^L--I^^B--C^^T--T1, heavyblue+linewidth(1.2));
draw(A--T--M^^D--K, orange);
draw(N--G^^A--L, purple);
draw(M--S^^N--S, red);
draw(L--S, red+dashed);
draw(circumcircle(A,G,F), cyan);
draw(circumcircle(M,F,I), magenta);
draw(S--T1, heavygreen+dashed);

dot("$A$", A, dir(120));
dot("$B$", B, dir(210));
dot("$C$", C, dir(330));
dot("$D$", D, dir(60));
dot("$M$", M, dir(90));
dot("$N$", N, dir(270));
dot("$I$", I, dir(330));
dot("$F$", F, dir(45));
dot("$K$", K, dir(210));
dot("$G$", G, dir(135));
dot("$S$", S, dir(150));
dot("$L$", L, dir(210));
dot("$T$", T, dir(225));
dot("$T'$", T1, dir(315));
[/asy]
Let $T = \overline{MI} \cap \overline{AF}$ be the $A$-mixtilinear intouch point, and let $S = \overline{NK} \cap (ABC)$. Since $\angle NSM = 90^\circ$, we just want $\overline{AG}$, $\overline{MS}$, $\overline{IF}$ concur. By (degenerate) Reim's we have $AGFI$ cyclic, so by radical axis it suffices to show $MIFS$ cyclic.

Let $T' = \overline{SF} \cap (ABC)$. Then $$(AN;TT') \overset{S}= (AK;TF) \overset{I}= (AD;\overline{TM} \cap \overline{AD}, \infty_{AD}) = \frac{TA}{TD},$$which implies $\overline{TT'} \parallel \overline{BC}$, from where it follows by Reim's that $MIFS$ is cyclic, as desired.
"which implies $\overline{TT'} \parallel \overline{BC}$". Why does it imply they are parallel?
Z K Y
The post below has been deleted. Click to close.
This post has been deleted. Click here to see post.
i3435
1350 posts
#5
Y by
If $\overline{TT'}||\overline{BC}$, then by angle bisector theorem $(AN;TT')=\frac{TA}{T'A}$. $\frac{TA}{TA'}=\frac{TA}{TD}$ since $TT'DA$ is an isosceles trapezoid.
Z K Y
The post below has been deleted. Click to close.
This post has been deleted. Click here to see post.
guptaamitu1
656 posts
#6 • 3 Y
Y by kamatadu, GeoKing, Om245
Let $T = \overline{AF} \cap \odot(O) \ne A$ (it is well known that $T \in \overline{MI}$), $X = \overline{ML} \cap \overline{NK}$, $Y = \overline{MF} \cap \odot(O) \ne M$, $Z = \overline{GI} \cap \odot(O) \ne G$ and $U = \overline{MT} \cap \overline{NG}$. By repeated application of Reims we obtain: points $\{A,G,F,I\},\{T,Y,F,I\}$ are concyclic ; $Y \in \overline{DI}$ and $\overline{TZ} \parallel \overline{BC}$.
[asy]
size(250);
pair A=dir(115),B=dir(-160),C=dir(-20), D = dir(65), N = dir(-90),M=dir(90),I = incenter(A,B,C), T = intersectionpoint(unitcircle,I--I+dir(M--I)*100),F=  extension(A,T,I,foot(I,M,N)),G = IP(F--3*F-2*N,unitcircle),L= extension(A,G,F,I),K = extension(D,I,A,F),X=extension(M,L,N,K),Y=IP(F--3*F-2*M,unitcircle),Z=IP(I--100*I-99*G,unitcircle),U=extension(M,T,N,G);
draw(unitcircle,cyan+linewidth(0.8));
dot("$A$",A,dir(A));
dot("$B$",B,dir(140));
dot("$C$",C,dir(C));
dot("$N$",N,dir(N));
dot("$M$",M,dir(M));
dot("$F$",F,dir(F));
dot("$T$",T,dir(T));
dot("$I$",I,dir(I));
dot("$G$",G,dir(G));
dot("$D$",D,dir(D));
dot("$K$",K,dir(210));
dot("$L$",L,dir(L));
dot("$X$",X,dir(-90));
dot("$Y$",Y,dir(Y));
dot("$Z$",Z,dir(Z));
dot("$U$",U,dir(20));
draw(A--D^^L--I^^B--C^^T--Z,green);
draw(B--A--C^^A--T--M,purple);
draw(M--Y--D,fuchsia);
draw(Z--G--N,fuchsia);
draw(circumcircle(I,F,Y)^^circumcircle(A,G,F),dotted+brown);
draw(A--L,fuchsia);
draw(L--M^^X--N,purple); 
draw(T--L,linewidth(0.7));
draw(L--U,dashed+linewidth(0.7));
[/asy]
By radical axes on $\odot(AGFI),\odot(TYFI),\odot(O) ~ \implies ~ \boxed{L \in \overline{TY}}$. Our problem is clearly equivalent to $X \in \odot(O)$, which by Pascal on $MXNGAT$ is further equivalent to points $K,L,U$ being collinear. By Ceva-Menelaus confi in $\triangle ILT$, this is equivalent to $(T,I ; U,M) = -1$. Indeed, $(T,I ; U,M) \stackrel{G}{=} (T,Z ; N,M) = -1$, completing the proof. $\blacksquare$
Z K Y
The post below has been deleted. Click to close.
This post has been deleted. Click here to see post.
crazyeyemoody907
450 posts
#7 • 5 Y
Y by v4913, eibc, Om245, Rounak_iitr, bjump
harmonics :heart_eyes:
China TST 2021/4/2 refactored wrote:
Scalene $\triangle ABC$ with incenter $I$ is inscribed in circle $\Omega$, with $M$, $N$ as the respective midpoints of arcs $\widehat{BAC}$, $\widehat{BC}$. Let $D$ be the reflection of $A$ in the perpendicular bisector of $\overline{BC}$, $E$ is the extouch point on $\overline{BC}$, and $F$ is defined so that $\overline{FI}\parallel\overline{BC}$ and $\angle BAF=\angle EAC$. Define $G=\overline{NF}\cap\Omega \enskip(\neq N)$, $L=\overline{AG}\cap\overline{IF}$, and $K=\overline{AF}\cap\overline{DI}$. Prove that $\overline{ML}, \overline{NK}$ meet on $\Omega$.
[asy]
//setup;
size(12cm);
pen blu,grn,blu1,blu2,lightpurple; blu=RGB(102,153,255); grn=RGB(0,204,0);
blu1=RGB(226,235,255); blu2=RGB(196,216,255); lightpurple=RGB(237,186,255); // blu1 lighter
//defn
pair M,N,I,A,X,P,Z; M=(7, 17.35); N=(7,-2.82); I=(2.59, 4.5); A=foot(M,N,I); X=foot(N,M,I); P=foot(I,M,N); Z=orthocenter(M,N,I);
pair reflect(pair P,pair A,pair B){return 2*foot(P,A,B)-P;}
pair F,G,L,D,G1,Z1,K; F=extension(A,X,Z,P); G=foot(M,N,F); L=extension(A,G,Z,I); D=reflect(A,M,N); G1=reflect(G,M,N); Z1=2*P-Z; K=extension(D,I,A,X);

//draw
filldraw(Z--M--N--cycle,blu1,blu); draw(circumcircle(A,M,N),blu);
draw(Z--Z1--M--X--A--N,blu);
draw(A--Z1^^Z--D--X^^D--extension(D,I,N,X)^^N--G, purple);
draw(A--L,magenta); draw(M--L^^N--foot(M,N,K),red);
//label
void pt(string s,pair P,pair v,pen a){filldraw(circle(P,.18),a,linewidth(.3)); label(s,P,v);}
pt("$M$",M,dir(90),blu); pt("$N$",N,dir(-90),blu); pt("$A$",A,dir(130),blu);
pt("$I$",I,dir(90),blu); pt("$X$",X,-dir(70),blu); pt("$P$",P,dir(130),blu);
pt("$Z$",Z,dir(130),blu); pt("$F$",F,dir(50),blu); pt("$G$",G,dir(130),purple);
pt("$L$",L,-dir(50),magenta); pt("$D$",D,dir(50),purple); pt("$G'$",G1,dir(50),purple);
pt("$Z'$",Z1,dir(-90),blu); pt("$K$",K,-dir(40),blu);
pt("",foot(M,N,K),(0,0),red); pt("",extension(D,I,N,X),(0,0),purple);
label("$\Omega$",(13, 16.7),blu);
[/asy]
Define:
  • $X=\overline{MI}\cap\overline{AF}\cap\Omega$, and $P$ as the foot from $I$ to $\overline{MN}$;
  • $Z=\overline{AM}\cap\overline{IP}\cap\overline{NX}$ as the orthocenter of $\triangle IMN$,
Note that:
  • By Ceva-Menelaus $(ZI;FP)=-1$ meanwhile $(F,\overline{DX}\cap\overline{MN};I,Z) \overset X=(AD;MN)=-1$, so $X,P,D$ collinear;
  • $(M,\overline{ZD}\cap\Omega;A,X)\overset Z=(AD;MN)=-1$ while $(MG;AX)\overset N=(PF;IZ)=-1$, so $Z,G,D$ also collinear;
Let $G',Z'$ be the respective reflections of $Z,G$ in $\overline{MN}$, so that $A,G',Z'$ collinear. Now we do yet more harmonic chasing:
\[(A,X;\overline{NK}\cap\Omega,D) \overset N=(I,\overline{NX}\cap\overline{DI};K,D) \overset X=(IZ;FP)=-1\text{ and}\]\[(A,X;\overline{ML}\cap\Omega,D)\overset M=(ZI;LZ')\overset A=(MN;GG')=-1,\]so the points coincide as desired!
This post has been edited 4 times. Last edited by crazyeyemoody907, Dec 16, 2022, 10:56 PM
Z K Y
The post below has been deleted. Click to close.
This post has been deleted. Click here to see post.
Mahdi_Mashayekhi
689 posts
#8
Y by
Obviously we need to prove $ML$ and $NK$ are concurrent on $ABC$. Let $AF$ and $AE$ meet $ABC$ at $S$ and $S'$. It's well known (I have proved them in "Korea winter program 2022 P6") that $S$ is $A$-mixtilinear intouch point so $M,I,S$ are collinear and $S'$ is reflection of $S$ about $MN$. Let $AE$ meet $MN$ at $X$.
Claim $: AGFI$ is cyclic.
Proof $:$ Note that $\angle GFI = \angle GFX = \angle 180 - \angle GMX = \angle 180 - \angle GMN = \angle 180 - \angle GAI$.
Claim $: PFIM$ is cyclic.
Proof $:$ Note that $\angle FIS = \angle ISS' = \angle MSS' = \angle MPS' = \angle MPF$.
Now By Radical Axis Theorem on $ABC$ and $AGFI$ and $PMIF$ we have that $AG,PM,IL$ are concurrent so $L,P,M$ are collinear. Now we only need to prove $N,K,P$ are collinear. Let $PK$ meet $ABC$ at $N'$.
$\frac{FK}{SK} . \frac{SA}{FA} = \frac{S'N'}{SN'} . \frac{\sin{ASP}}{\sin{AFP}} . \frac{\frac{SA}{AP}}{\frac{FA}{AP}} = \frac{S'N'}{SN'} . \frac{\sin{ASP}}{\sin{AFP}} . \frac{\frac{\sin{APS}}{\sin{ASP}}}{\frac{\sin{APF}}{\sin{AFP}}} = \frac{S'N'}{SN'} . \frac{AS}{AS'}$ Also Note that $\frac{SA}{SN} . \frac{S'N}{S'A} = \frac{SA}{S'A} = \frac{SA}{SD} = \frac{SA}{SK} . \frac{SK}{SD} = \frac{SA}{SK} . \frac{KI}{ID} = \frac{SA}{SK} . \frac{FK}{FA}$ so $\frac{SA}{SN} . \frac{S'N}{S'A} = \frac{S'N'}{SN'} . \frac{AS}{AS'}$ so $\frac{S'N}{SN} = \frac{S'N'}{SN'}$ so $N'$ is $N$ which proves that $P,K,N$ are collinear.
we're Done.
This post has been edited 3 times. Last edited by Mahdi_Mashayekhi, Dec 31, 2022, 11:05 AM
Z K Y
The post below has been deleted. Click to close.
This post has been deleted. Click here to see post.
Shayan-TayefehIR
104 posts
#10 • 1 Y
Y by Mahdi_Mashayekhi
Lemma :Let $ABCD$ be a cyclic quadrilateral which its diagonals $AC$ and $BD$ intersect at point $R$ and also $P , Q$ are intersection points of $AD , BC$ and $AB , CD$ respectively. So the Miquel point of quadrilateral $ABCD$ , is the foot of altitude from point $R$ to the line $PQ$.

Proof :Let $S$ be the Miquel point of quadrilateral $ABCD$ , then since $\angle ASQ + \angle ASP=\angle ABC + \angle ADC =180$ , points $P , S , Q$ are collinear and if $O$ be the center of circumcircle of $ABCD$ , one can see that :
$$\angle ASC=\angle ASP - \angle PSB=\angle ABC - \angle ADC=180 - 2\angle ADC=180-\angle AOC$$Thus quadrilaterals $SAOC$ and $SBOD$ are cyclic and by concurrency of radical axes of circles , lines $AC , BD , OS$ are concurrent at point $R$. As the result , since $PQ$ is polar of $R$ wrt the circumcircle of $ABCD$ , we have $OS \perp PQ$ and we're done.


Firstly , Since $AF$ passes trough the $A-$mixtilinear point of triangle $\triangle ABC$ , name that as $P$ , we know that points $M , I , P$ are collinear. Now , by angle chasing one can see that :
$$\angle AGF=\angle AGN=\angle ABN=\angle B + \frac{\angle A}{2}=180 - \angle AIF$$Thus quadrilateral $AGFI$ is cyclic and by Lemma , point $T$ , the second intersection point of line $LN$ and circle $\omega$ is the Miquel point of this quadrilateral. Now since quadrilateral $TFIN$ is cyclic , we can get :
$$\angle FTN = \angle AIF = \angle B + \frac{\angle A}{2} = \angle ACN = \angle DTN$$So points $D , F , T$ are collinear. Now if line $DI$ intersects the circle $\omega$ at point $X$ for second time , then applying Pascal Theorem on $MXDAPM$ , gives us that points $M , K , X$ are collinear and also again by Pascal on $XDTAPM$ , points $P , X , L$ are collinear too. So note that by concurrency of $AG , IF , PX$ and since quadrilateral $AGFI$ is cyclic , thus $IFXP$ is cyclic too and by Lemma , point $S$ , the second intersection point of line $ML$ and circle $\omega$ , is the Miquel point of this quadrilateral and hence , $KS \perp ML$ and $NK$ is perpendicular to the line $ML$ at point $S$ , so we're done.
This post has been edited 2 times. Last edited by Shayan-TayefehIR, Jun 8, 2024, 1:30 PM
Z K Y
The post below has been deleted. Click to close.
This post has been deleted. Click here to see post.
kamatadu
465 posts
#11 • 2 Y
Y by GeoKing, HoripodoKrishno
This is one of the hardest geos I have ever solved (alright, after looking at the other solns in the thread, it seems I extended the config wayy too much :wallbash_red: ). :stretcher: Also lol, I literally had to take a 2 hour anime (``Samurai Champloo") break and then finally after coming back was I able to get rid of the clog in my head and finally solve the problem :rofl: some rant about the anime

https://i.imgur.com/6X3UVuK.png

Let $Z=MN\cap IF$, $P=\odot(IFT)\cap\odot(ABC)$, $Q=\odot(MIF)\cap\odot(ABC)$, $X=AM\cap IF$, $Y=NG\cap DI$, $J=MT\cap KL$, $T$ be the $A$-mixtilinear intouch point and $T'=AE\cap\odot(ABC)$. :stretcher:

Firstly it is well known that $\overline{M-I-T}$ are collinear. Let $M'=PF\cap\odot(ABC)$. Now Reim's on $\left\{\odot(PFIT),\odot(ABC)\right\}$ with lines $\left\{TI,PF\right\}$ gives that $IF\parallel MM'$ and as $IF\parallel BC$, we get that $M'\equiv M$ which gives that $\overline{P-F-M}$ are collinear. Let $A'=TF\cap\odot(ABC)$. Now this time Reim's on $\left\{\odot(PFIT),\odot(ABC)\right\}$ with lines $\left\{TF,PI\right\}$ gives that $IF\parallel A'D$ which gives $A'\equiv A$ and so $\overline{A-F-T}$ are also collinear. Now the from the Diameter of Incircle Lemma, we have that $\overline{A-Z-E-T}$ are collinear (can be derived from simple length computation and then showing that $\operatorname{dist}(Z,BC)=\operatorname{dist}(BC)$).

Now note that the $A$-nagel cevian and the $A$-symmedian are isogonal conjugates ($\sqrt{bc}$ Inversion proves the fact). This gives that $TT'\parallel BC$ and so $AN$ is the angle-bisector of $\angle TAT'$. Let $T''=QF\cap\odot(ABC)$ Now again ( :rotfl: ) Reim's on $\left\{\odot(MIFQ),\odot(ABC)\right\}$ with lines $\left\{QF,MI\right\}$ gives that $TT''\parallel BC\implies T''\equiv T'$.

So $F$ is basically the intersection of the $A$-symmedian with $IF$. Now we begin with the actual solution. :rotfl: :stretcher:

Now, $MN\perp BC$ and $IF\parallel BC\implies MN\perp IF\implies\measuredangle MZI=90^\circ=\measuredangle MAN=\measuredangle MAI\implies MAIZ$ is cyclic and similarly $NTIZ$ is also cyclic. Also, $\measuredangle AGF=\measuredangle AGN=\measuredangle ABN=\measuredangle (AN,BC)=\measuredangle AIF\implies AGFI$ is also cyclic. Finally, $\measuredangle T'PI=\measuredangle T'PD=\measuredangle T'TD=\measuredangle AT'T=\measuredangle AZI=\measuredangle T'ZI\implies PIZT'$ is also cyclic. Now Radax on $\left\{\odot(ABC),\odot(MAIZ),\odot(NITZ)\right\}$ gives that $X=AM\cap IF\cap NT$. Also, Radax on $\left\{\odot(ABC),\odot(MZIA),\odot(PIZT')\right\}$ gives $X=AM\cap IF\cap NT\cap PT'$. Then Radax on $\left\{\odot(ABC),\odot(MIFQ),\odot(AGFI)\right\}$ gives $L=AG\cap IF\cap MQ$ and then another Radax on $\left\{\odot(ABC),\odot(MIFQ),\odot(PFIT)\right\}$ finally also gives $L=AG\cap IF\cap MQ\cap PT$. :stretcher:

Now $\measuredangle MAI=90^\circ$ simply means that $MA$ is the external-angle-bisector of $\angle BAC$ which is same as the external-angle-bisector of $\angle FAZ$ from the fact that $AN$ is the internal-angle-bisector of $\angle TAT'$. Combining these two pieces of information, we get that \[-1=(X,I;F,Z)\overset{T'}{=}(P,G;Q,A).\]Now finally we also have that $(M,J;I,T)=-1$ (due to the complete quadrilateral of $PFIT$). Now \[-1=(X,I;F,Z)\overset{N}{=}(T,I;NG\cap MT,M),\]which finally gives that $J=LK\cap MT\cap NG$. Now then \[-1=(T,I;J,M)\overset{F}{=}(K,I;Y,P)=(P,Y;K,I)\overset{N}{=}(P,G;NK\cap\odot(ABC),A),\]which on combining with the result we derived at the end of the previous paragraph finishes the proof.




Remark: There are many missing cyclic quads in the diagram which I decided not to show because it was just making the diagram way too complex.


More Remark: Bro, this application of converse of Pascal is sooo cleverrr!!!!! Arjun bro ORZ. :omighty: The quote mentioned below follows from the fact that the 6 points lie on a conic and as 5 points determine a conic, which are already on a circle, the conic is itself the circle and thus the 6th point lies on the circle. Thank you soo much for this lesson!! :love:
guptaamitu1 wrote:
By radical axes on $\odot(AGFI),\odot(TYFI),\odot(O) ~ \implies ~ \boxed{L \in \overline{TY}}$. Our problem is clearly equivalent to $X \in \odot(O)$, which by Pascal on $MXNGAT$ is further equivalent to points $K,L,U$ being collinear.
This post has been edited 2 times. Last edited by kamatadu, Jun 8, 2023, 8:11 AM
Reason: Arjun bro ORSMAXXXX
Z K Y
The post below has been deleted. Click to close.
This post has been deleted. Click here to see post.
popop614
270 posts
#12
Y by
This problem is great. I like it. Solved after 6 hours of english project work (despair)

Observe that the condition is equivalent to showing that $ML$ and $NK$ are concurrent on $\omega$, the circumcircle of $ABC$.

Define $T$ as the mixtilinear intouch point (hence collinear with $A$, $F$, and $K$) and $T'$ the reflection of $T$ over the perpendicular bisector of $BC$. A quick angle chase left as an exercise to the reader reveals that $AGIF$ is cyclic, whence $\measuredangle AGI$ = $\measuredangle AFI$ = $\measuredangle ATT'$, revealing that $G$, $I$, and $T'$ are collinear.

Let $X = \overline{NK} \cap \omega \neq N$. Then pascal's on cyclic hexagon $MXNAGT$ reveals that $\overline{MX} \cap \overline{AG}$, $\overline{XN} \cap \overline{GT}$, and $\overline{NA} \cap \overline{MT}=I$ are collinear, so if $\overline{XN} \cap \overline{GT}$ lies on line $IF$ we're done.

Let $P = \overline{DI} \cap \omega \neq D$. It can easily be proven that $PFIT$ is cyclic, whence $PT$, $IF$, and $AG$ concur. Now pascal's on $GTMPAN$ shows that $\overline{GT} \cap \overline{PA}$, $\overline{TM} \cap \overline{AN} = I$, and $\overline{MP} \cap \overline{GN} = F$ are collinear so we have that $GT$ and $AP$ concur on $IF$.

Observe that
\[ -1 = (A, D; M, N) \overset{I}{=} (N, P; T, A). \]Furthermore if $XGAP$ is harmonic then the problem is completed (uniqueness of harmonic conjugate, perspectivity thru AP cap GT, which lies on IF). Now observe:
\[ (X, A; G, P) \overset{N}{=} (K, A; F, \overline{NP} \cap \overline{AT}) = \overset{P}{=} (D, A; \overline{PF}\cap\omega, N), \]and in particular if $P$, $F$, and $M$ are collinear this is enough.

Indeed, notice that \[ \measuredangle MPT = \measuredangle MAT = \measuredangle ATM + \measuredangle TMA = \measuredangle MTD + \measuredangle TMA = \measuredangle MAD + \measuredangle TMA = \measuredangle FIM = \measuredangle FIT = \measuredangle FPT, \]so $M$, $F$, and $P$ are collinear, as desired.
This post has been edited 1 time. Last edited by popop614, Jun 8, 2023, 11:16 AM
Reason: oops
Z K Y
The post below has been deleted. Click to close.
This post has been deleted. Click here to see post.
YaoAOPS
1501 posts
#13 • 2 Y
Y by GeoKing, megahertz13
Solved this while gradually losing my sanity.

Note that it is equivalent to show that $\overline{ML} \perp \overline{NK}$.
Let $T$ be the $A$-mixtilinear touch point. Then it is well-known that $AT$ is isogonal to $AE$, so we can delete $E$.
Let $P = \overline{IF} \cap \overline{MN}$.
Note that quadrilaterals $AMPI$, $TNPI$ are cyclic due to right angles.

Claim: $P$ lies on $TD$.
Proof. Angle chase to get \[ \measuredangle TPN = \measuredangle TIN = \measuredangle MIA = \measuredangle MPA = \measuredangle DPM = \measuredangle DPN \]. $\blacksquare$

Claim: $AM$, $IP$, $TN$, $GD$ concur at some point $R$.
Proof. Quadrilateral $GIPD$ is cyclic since \[ \measuredangle GIP = \measuredangle GIF = \measuredangle GAF = \measuredangle GAT = \measuredangle GDT = \measuredangle GDP \]then radical axis on $(AMPI)$, $(TNPI)$, $\Omega$ and $(GIPD)$ gives the result. are cyclic due to right angles. $\blacksquare$

Claim: Let $Q = \overline{GD} \cap \overline{AT}$. Then, $L$, $Q$, $M$ are collinear.
Proof. Note that $\overline{GF}$, $\overline{RT}$, $\overline{AI}$ concur at $N$. Then Desargues' on $\triangle GRA$ and $\triangle FTI$ gives the result. $\blacksquare$
Note that $AMDN$ is a harmonic quadrilateral.

Claim: $S = \overline{MF} \cap \overline{KID}$ lies on $\Gamma$.
Proof. Note that \[ (NM;AD) \overset{I}= (A,T;N,\Gamma \cap \overline{KID}) = -1 \]and that \[ (NM;AD) \overset{T}= (RI;FP) \overset{M}= (A,T;\Gamma \cap \overline{MF},N) = -1 \]$\blacksquare$

Claim: $\overline{MQL} \cap \overline{NK}$ lies on $\Gamma$.
Proof. Follows as
\[
    (A,T;D, \Gamma \cap \overline{MQL}) \overset{Q}= (TA;GM) \overset{N}= (RI;FP) = -1
  \]and that \[ (A,T;D, \Gamma \cap \overline{NK}) \overset{K}= (AT;SN) = -1 \]$\blacksquare$
Attachments:
This post has been edited 2 times. Last edited by YaoAOPS, Jul 29, 2023, 1:12 PM
Z K Y
The post below has been deleted. Click to close.
This post has been deleted. Click here to see post.
eibc
598 posts
#14 • 1 Y
Y by Shreyasharma
I never want to touch a harmonic bundle again :(

[asy]
unitsize(3cm);

pair A = dir(120); pair B = dir(205); pair C = dir(335); pair I=incenter(A, B, C); pair M=dir(90); pair N=dir(270); pair D=dir(60); pair T=IP(Line(I, M, 10), unitcircle, 0); pair TTT =foot(T, M, N); pair P=foot(I, M, N); pair TT=2*TTT-T; pair F= extension(A,T,I,P); pair G=IP(Line(N, F, 10), unitcircle, 1); pair K=extension(A, F, D, I); pair L=extension(A, G, I, F); pair X=extension(A, M, G, D); pair Q=IP(Line(M, F, 10), unitcircle, 1); pair Y=extension(A, T, G, D); pair Z=extension(L, M, N, K);

draw(unitcircle, heavygreen); draw(A--B--C--cycle, heavygreen); draw(A--T, heavygreen); draw(N--G, heavygreen); draw(A--L, heavygreen); draw(A--K--D, heavygreen); draw(M--N, heavygreen); draw(N--Z, heavygreen); draw(M--L, heavygreen); draw(M--X--D, red); draw(P--X--N, red); draw(A--TT, red); draw(D--T, red); draw(M--Q, red); draw(D--Q, red); draw(A--N, heavygreen); draw(M--T, heavygreen);

dot("$A$", A, dir(A)); dot("$B$", B, dir(B)); dot("$C$", C, dir(C)); dot("$I$", I, N); dot("$M$", M, dir(M)); dot("$N$", N, dir(N)); dot("$D$", D, dir(D)); dot("$T$", T, dir(T));  dot("$T'$", TT, dir(TT)); dot("$F$", F, NW); dot("$G$", G, dir(G)); dot("$K$", K, dir(K)); dot("$L$", L, NW); dot("$X$", X, dir(X)); dot("$P$", P, E); dot("$Q$", Q, dir(Q)); dot("$Y$", Y, SE); dot("$Z$", Z, dir(Z));
[/asy]

Let $T$ be the $A$-mixtilinear touch point; it's well-known that $AT$ and $AE$ are isogonal (consider a $\sqrt{bc}$ inversion), so we can ignore $E$ from now on. It's also well-known that $T$, $I$, and $M$ are collinear. Additionally, denote $T'$ as the reflection of $T$ over $MN$.

Claim: $AM$, $GD$, $FI$, and $TN$ are concurrent.
Proof: Let $X = \overline{MA} \cap \overline{NT}$. By Pascal's on $MMANNT$, we find that $X$ lies on line $FI$. Then by Pascal's on $NNGDAT$ we see that $X$ lies on line $GD$, too. From this point onwards, refer to line $FI$ as $\ell$.

Claim: $\ell$, $MN$, and $TD$, and $T'A$ are concurrent
Proof: By Pascal's on $AMNGDT$ we find that $\overline{MN} \cap \overline{DT}$ lies on $\ell$. Since $\overline{DT}$ and $\overline{AT'}$ are reflections across $\overline{MN}$, this concurrency point, say $P$, lies on $\overline{AT'}$ too.

Claim: $MF$ and $DK$ meet on $\Omega$
Proof:Let $Q = \overline{MF} \cap \Omega$ and $Q' = \overline{DIK} \cap \Omega$. Note that $\angle TAN = \angle NAT'$ and $\overline{XA} \perp \overline{AI}$, so by the right angles and angle bisectors lemma we have $(X, I; F, P) = -1$. Also, $AMDN$ is a harmonic quadrilateral from symmetry. Therefore, we get
$$\begin{aligned} -1 &= (X, I; F, P) \overset{M}{=} (A, T; Q, N), \\ -1 &= (N, M; D, A) \overset{I}{=} (A, T; Q', N). \end{aligned}$$Therefore $Q \equiv Q'$, as needed.

Claim: $AT$, $GD$, and $LM$ are concurrent
Proof: Since $AD$ and $FX$ concur at the point at infinity along line $\ell$, this follows by Descargue's on $\triangle AMD$ and $\triangle FLX$. Denote this concurrency point as $Y$.

Claim: (problem statement lol) $LM$ and $NK$ concur on $\Omega$
Proof: Let $Z = \overline{LM} \cap \Omega$ and $Z' = \overline{NK} \cap \Omega$. Then from our earlier harmonic bundles we have
$$\begin{aligned} -1 &= (A, T; Q, N) \overset{F}{=} (T, A, M, G) \overset{Y}{=} (A, T, Z, D), \\ -1 &= (T, A; N, Q) \overset{K}{=} (A, T; Z', D), \end{aligned}$$and this implies $Z \equiv Z'$, so we're finally done.
This post has been edited 4 times. Last edited by eibc, Sep 17, 2023, 9:02 PM
Reason: added diagram
Z K Y
The post below has been deleted. Click to close.
This post has been deleted. Click here to see post.
GrantStar
815 posts
#15 • 4 Y
Y by ihatemath123, OronSH, megahertz13, bjump
I mean its not thattt bad.
https://cdn.artofproblemsolving.com/attachments/5/5/7e801583a46a34a8a894d59d296d66aa00227b.jpeg

Let $T$ be the $A$-mixtilinear touch point.

Claim: $\angle TAB=\angle CAE$, meaning $T$ lies on line $A-F-K.$
Proof: Consider a $\sqrt{bc}$ inversion. Notice that it swaps lines $AB$ and $AC,$ and sends $\Omega$ to line $BC$. Thus, it sends the $A$-mixtillinear incircle to the $A$-excircle and sends $T$ to $E,$ proving the claim.

Claim: Quadrilateral $GAMT$ is harmonic.
Proof: Call line $I-F-L$ line $\ell.$
  • Pascal on $NNTMMA$ gives $X=AM \cap NT$ lies on $\ell.$
  • Pascal on $ATMGNA$ lives $Y=AA\cap GM$ is on $\ell.$
  • Pascal on $AANTTM$ gives $AA\cap TT$ is on $\ell.$
Thus, $AA,TT,MG$ intersect on $\ell$ impplying the claim.

Now, projecting this bundle onto $\ell$ from $N$ gives $(X,I;F,J=MN\cap \ell).$ But as $\angle XTI=\angle MTN=90,$ ray $TI$ bisects $\angle ATJ$ hence $T,J,D$ are collinear.

Then, pascal on $DGNMAT$ gives $X$ is on line $DG.$ Also, Pascal on $MMHDAT$ where $H$ is the intersection of $MF$ with $\Omega$ gives $H,I,D$ collinear.

Now, let $P=NK\cap \Omega \neq N.$ Then,
\[-1=(AT;GM)\overset{F}{=}(TA;HN)\overset{K}{=}(AT;PD)\]Now, let $X'$ be the reflection of $X$ about $J$ which by symmetry lies on line $MD.$ Then, lines $AX'$ and $XD=CD$ are symmetric about $MN.$ Thus, if $AX'$ hits $\Omega$ at $Y\neq A,$ we have $\angle BAG=\angle CAX'$ so
\[\angle LAI=\angle GAB+\angle BAI=\angle CAI+\angle CAX'=\angle IAX'.\]Then, as $MN$ is a diameter, $\angle XAI=90$ so $-1=(XI;LX').$ Now, from the above harmonic bundle we have $-1=(AT;PD)\overset{M}{=}(X,I;MP\cap \ell,X')$ implying $M,P,L$ collinear. Thus lines $ML,NK$ intersect at $P,$ which is on $\Omega$ as desired.
This post has been edited 2 times. Last edited by GrantStar, Aug 29, 2023, 10:55 PM
Z K Y
The post below has been deleted. Click to close.
This post has been deleted. Click here to see post.
MathLuis
1471 posts
#16 • 3 Y
Y by crazyeyemoody907, Shreyasharma, bjump
A really nice projective practice problem, got it in 24 mins, i give it a 9/10 since at the end it gets a little messy.
Let $AF \cap (ABC)=T_A$ and $AE \cap (ABC)=T_A'$, let $LM \cap (ABC)=Q$, now before going for the claims notice some trivial propeties, first $T_A$ is the A-mixtilinear intouch point of $\triangle ABC$ by $\sqrt{bc}$ inversion and by the same it holds that $M,I,T_A$ are colinear, also $T_AT_A' \parallel BC$ so $DT_A=AT_A'$.
Claim 1: $AGFI$ is cyclic.
Proof: This follows trivially by Reim's on $(ABC)$ and $FI$ as $N$ is the midpoint of arc $BC$ in $(ABC)$ so its tangent to $(ABC)$ is parallel to $BC$.
Claim 2: $Q,F,T_A'$ are colinear.
Proof: By PoP we get $LF \cdot LI=LA \cdot LG=LQ \cdot LM$ so $QFIM$ is cyclic hence by Reim's on $(QFIM), (ABC)$ we get $Q,F,T_A'$ colinear as desired.
A projective Finishing: We do some projections to show that $Q,K,N$ colinear:
$$(A, K; T_A, F) \overset{I}{=} (A, D; IT_A \cap AD, \infty_{BC}) = \frac{T_AA}{DA}=\frac{T_AA}{AT_A'} = (A, N; T_A, T_A') \implies NK, T_A'F \; \text{meet at} \; Q$$Thus we are done :D.
This post has been edited 2 times. Last edited by MathLuis, Sep 18, 2023, 12:44 AM
Z K Y
The post below has been deleted. Click to close.
This post has been deleted. Click here to see post.
Shreyasharma
667 posts
#17 • 1 Y
Y by bjump
Projective has scarred me.

[asy]
/* Geogebra to Asymptote conversion, documentation at artofproblemsolving.com/Wiki go to User:Azjps/geogebra */
import graph; size(15cm);
real labelscalefactor = 0.5; /* changes label-to-point distance */
pen dps = linewidth(0.7) + fontsize(10); defaultpen(dps); /* default pen style */
pen dotstyle = black; /* point style */
real xmin = -4.212913392833375, xmax = 5.63653035739966, ymin = -4.15652363665763, ymax = 3.77358934182421; /* image dimensions */
pen wwqqcc = rgb(0.4,0,0.8); pen ubqqys = rgb(0.29411764705882354,0,0.5098039215686274); pen zzttff = rgb(0.6,0.2,1); pen qqzzcc = rgb(0,0.6,0.8); pen qqttcc = rgb(0,0.2,0.8);

draw(circle((0.5443167447003234,-0.4949349743389013), 3.172400915766288), linewidth(0.9) + zzttff);
draw(circle((-0.7939325706933222,0.8509994261452113), 1.3735614089382795), linewidth(0.9) + blue);
draw(circle((-0.7953501608494566,1.3634582677774578), 1.8702555618022134), linewidth(0.9) + qqzzcc);
/* draw figures */
draw((-1.1764612326917248,2.1702199410229266)--(-2.2922035259762987,-1.9155969061141573), linewidth(0.9) + wwqqcc);
draw((-2.2922035259762987,-1.9155969061141573)--(3.388653361662767,-1.8998822259328612), linewidth(0.9) + wwqqcc);
draw((3.388653361662767,-1.8998822259328612)--(-1.1764612326917248,2.1702199410229266), linewidth(0.9) + ubqqys);
draw((0.5355411187616752,2.677453803655398)--(0.5530923706389715,-3.667323752333202), linewidth(0.9));
draw((0.5355411187616752,2.677453803655398)--(-1.192683562367836,-3.1495455867639897), linewidth(0.9));
draw((-1.1844998230464998,-0.4658635756563433)--(-0.3960858022931656,-0.46368262400914606), linewidth(0.9) + blue);
draw((-1.1764612326917248,2.1702199410229266)--(-3.8350570442277574,-0.4731956841492926), linewidth(0.9) + blue);
draw((-3.8350570442277574,-0.4731956841492926)--(0.5355411187616752,2.677453803655398), linewidth(0.9) + blue);
draw((-2.4626528494792534,0.5161344743313137)--(0.553092370638971,-3.6673237523332007), linewidth(0.9));
draw((-1.192683562367836,-3.1495455867639897)--(-2.4626528494792534,0.5161344743313137), linewidth(0.9));
draw((2.2959769984063434,-3.1398950734597455)--(-2.4626528494792534,0.5161344743313137), linewidth(0.9));
draw((-1.192683562367836,-3.1495455867639897)--(0.553092370638971,-3.6673237523332007), linewidth(0.9) + qqttcc);
draw((0.553092370638971,-3.6673237523332007)--(2.2959769984063434,-3.1398950734597455), linewidth(0.9) + qqttcc);
draw((2.2959769984063434,-3.1398950734597455)--(0.5355411187616752,2.677453803655398), linewidth(0.9));
draw((-1.192683562367836,-3.1495455867639897)--(2.2959769984063434,-3.1398950734597455), linewidth(0.9) + zzttff);
draw((-3.8350570442277574,-0.4731956841492926)--(-1.1844998230464998,-0.4658635756563433), linewidth(0.9) + blue);
draw((-1.1869019636037623,-1.253594116494785)--(2.2503235210467407,2.179699290269044), linewidth(0.9) + wwqqcc);
draw((-1.1764612326917248,2.1702199410229266)--(-1.1869019636037623,-1.253594116494785), linewidth(0.9) + wwqqcc);
draw((-1.1869019636037623,-1.253594116494785)--(-1.192683562367836,-3.1495455867639897), linewidth(0.9) + qqttcc);
draw((-1.1764612326917248,2.1702199410229266)--(2.2959769984063434,-3.1398950734597455), linewidth(0.9) + qqttcc);
/* dots and labels */
dot((-1.1764612326917248,2.1702199410229266),dotstyle);
label("$A$", (-1.2543352321169878,2.284937672900228), NE * labelscalefactor);
dot((-2.2922035259762987,-1.9155969061141573),dotstyle);
label("$B$", (-2.4901969954542125,-2.0686662645189635), NE * labelscalefactor);
dot((3.388653361662767,-1.8998822259328612),dotstyle);
label("$C$", (3.4644096824433257,-2.0031281407298573), NE * labelscalefactor);
dot((-0.3960858022931656,-0.46368262400914606),linewidth(4pt) + dotstyle);
label("$I$", (-0.3461640878464513,-0.6268275411586292), NE * labelscalefactor);
dot((0.5355411187616752,2.677453803655398),linewidth(4pt) + dotstyle);
label("$M$", (0.4871063434945564,2.7717923067621593), NE * labelscalefactor);
dot((-1.192683562367836,-3.1495455867639897),linewidth(4pt) + dotstyle);
label("$T$", (-1.3011481776979432,-3.3700661511883565), NE * labelscalefactor);
dot((2.2959769984063434,-3.1398950734597455),linewidth(4pt) + dotstyle);
label("$T'$", (2.3596241667327758,-3.323253205624709), NE * labelscalefactor);
dot((-1.1844998230464998,-0.4658635756563433),linewidth(4pt) + dotstyle);
label("$F$", (-1.1232589844903125,-0.3646750460022047), NE * labelscalefactor);
dot((0.553092370638971,-3.6673237523332007),linewidth(4pt) + dotstyle);
label("$N$", (0.5151941108431297,-3.866283374163017), NE * labelscalefactor);
dot((-2.110941040256875,1.241075874065847),linewidth(4pt) + dotstyle);
label("$G$", (-2.2842200348980084,1.3018658160636365), NE * labelscalefactor);
dot((2.2503235210467407,2.179699290269044),dotstyle);
label("$D$", (2.284723453803247,2.2755750837874986), NE * labelscalefactor);
dot((-3.8350570442277574,-0.4731956841492926),linewidth(4pt) + dotstyle);
label("$L$", (-4.016299021393362,-0.6551138826931464), NE * labelscalefactor);
dot((-1.1869019636037623,-1.253594116494785),linewidth(4pt) + dotstyle);
label("$K$", (-1.476048890627472,-1.4571094919515259), NE * labelscalefactor);
dot((-2.4626528494792534,0.5161344743313137),linewidth(4pt) + dotstyle);
label("$X$", (-2.6680861886618437,0.49668315236890426), NE * labelscalefactor);
clip((xmin,ymin)--(xmin,ymax)--(xmax,ymax)--(xmax,ymin)--cycle);
/* end of picture */
[/asy]

Let $T = AF \cap \Omega$. Note that $T$ is the $A$-mixintillinear touch point. Thus the points $M$, $I$ and $T$ are collinear. Furthermore by Reims on antiparallel $AG$ and $NN$ we find that $AGFI$ is cyclic. Now let $T'$ be the reflection of $T$ about $MN$.

We will now let $X = NK \cap \Omega$. Then $X$, $F$ and $T'$ are collinear. To see this let $F' = XT' \cap AT$. Now we will show that $\frac{KF'}{KA} = \frac{KI}{KD}$. Consider projecting,
\begin{align*}
(F'T,KA) \overset{X}{=} (T'T, NA) \implies \frac{F'K}{F'A} \cdot \frac{TA}{TK} = \frac{T'N}{T'A} \cdot \frac{TA}{TN}
\end{align*}However this reduces to,
\begin{align*}
\frac{F'K}{F'A} = \frac{TK}{T'A} = \frac{TK}{TD} = \frac{KI}{KD}
\end{align*}as desired.

Now by Reims on antiparallel $TT'$ and $AG$ we find that the points $M$, $I$, $F$ and $X$ are concyclic. Therefore by radical axis on $(AGFI)$, $(MXFI)$ and $(ABC)$ we indeed have $AG$, $MX$, and $IF$ are concurrent. However this means we are done.
This post has been edited 5 times. Last edited by Shreyasharma, Nov 21, 2023, 8:28 AM
Z K Y
The post below has been deleted. Click to close.
This post has been deleted. Click here to see post.
shendrew7
793 posts
#18
Y by
Let $T$ be the mixtilinear incircle touch point and $U$ be the reflection of $T$ over $MN$. It is well known that $T$ lies on $AK$, $U$ lies on $AE$, and $TI$ passes through $M$.

Denote $X = LM \cap (ABC)$. Note $AIFG$ is cyclic, as
\[\measuredangle GFI = \measuredangle GNN = \measuredangle GAI.\]
Power of a point also gives us another cyclic quadrilateral $XMIF$, and
\[\measuredangle MXF = \measuredangle MIF = \measuredangle TNM = \measuredangle MNU = \measuredangle MXU,\]
so $X$, $F$, and $U$ are collinear. As $TI$ bisects $\angle ATD$, if we let $P = TI \cap AD$, we use angle bisector theorem to get
\[(AN;UT) = \frac{UA}{TA} = \frac{TD}{TA} = \frac{PD}{PA} = (AD; \infty P) = (AK;FT).\]
Thus Prism lemma tells us $FU$ and $NK$ meet on the circumcircle, or $X$, as desired. $\blacksquare$
Z K Y
The post below has been deleted. Click to close.
This post has been deleted. Click here to see post.
EthanWYX2009
842 posts
#19 • 1 Y
Y by ys-lg
Sketch
Z K Y
The post below has been deleted. Click to close.
This post has been deleted. Click here to see post.
Markas
105 posts
#20 • 1 Y
Y by GeoKing
Let T be the A-mixtillinear touch point. It is known that AT and AE are isogonal. Also $T \in IM$. Let T' be the reflection of T across MN. We will now prove that AM, GD, FI and TN are concurrent. Let $MA \cap NT = X$. By Pascal on MMANNT we get that $X \in FI$. By Pascal on NNGDAT we get that $X \in GD$ $\Rightarrow$ AM, GD, FI and TN are concurrent at X. Now we will show that FI, MN, TD and T'A are concurrent. By Pascal on AMNGDT we get that $MN \cap DT \in FI$. Since $S_{AD} \equiv S_{TT'} \equiv MN$ and AD = TT', T'A, TD and MN are concurrent $\Rightarrow$ FI, MN, TD and T'A are concurrent in H for example. Now we will show that $MF \cap DK \in \Omega$ by phantom points. Let $MF \cap \Omega = Y$ and $DK \cap \Omega = Y'$. Now $\angle TAN = \angle NAT'$ and $AX \perp AI$ $\Rightarrow$ by right angle and bisectors we have that $(X,I;F,H) = -1$. Also AMDN is harmonic, since it is a kite. Now projecting we get $(X,I;F,H)\stackrel{M}{=}(A,T;Y,N) = -1$. Also $(N,M;A,D)\stackrel{I}{=}(A,T;Y',N) = -1$ $\Rightarrow$ $(A,T;Y,N) = (A,T;Y',N) = -1$ $\Rightarrow$ $Y \equiv Y'$ $\Rightarrow$ $MF \cap DK \in \Omega$ (at Y). Now we will show that AT, GD and LM are concurrent. This directly follows from Desargues theorem for $\triangle AMD$ and $\triangle FLX$ and let $AT \cap GD \cap LM = J$. Lastly we want to show that $LM \cap NK \in \Omega$ by phantom points. Let $LM \cap \Omega = P$ and $NK \cap \Omega = P'$. Now projecting we get $(A,T;Y,N)\stackrel{F}{=}(T.A;M,G)\stackrel{J}{=}(A,T;P,D) = -1$ and $(T,A;N,Y)\stackrel{K}{=}(A,T;P',D) = -1$ $\Rightarrow$ $(A,T;P,D) = (A,T;P',D) = -1$ $\Rightarrow$ $P \equiv P'$, which is what we wanted to initially prove $\Rightarrow$ we are ready.
Z K Y
The post below has been deleted. Click to close.
This post has been deleted. Click here to see post.
awesomehuman
496 posts
#21 • 2 Y
Y by v4913, crazyeyemoody907
Let $P$ be the intersection of $MN$ and $IF$. Let $T$ be the $A$-mixtilinear touch point. It is well known that $F$, $I$, and $P$ lie on $TA$, $TM$, and $TD$ respectively.

well known stuff

Let $X$ be the intersection of $(FIM)$ and $(ABC)$. We will show $X$ is on $ML$ and $NK$. By inversion about $N$, $AGFI$ is cyclic. By the radical center theorem, $AG$, $FI$, and $MX$ concur. Therefore, $X$ is on $ML$.

Let $I'$, $F'$, and $T'$ be the reflections of $I$, $F$, and $T$ about $MN$.

Claim: $NK$ and $T'F$ intersect on $(ABC)$
Let $R$ be the exsimilicenter of $FI$ and $II'$. By Monge's theorem on $AD$, $FI$, and $II'$, $R$ is on $NK$. Because $(RP;II')=(FI';PF')$ and $A=NI\cap T'P$, $M=NP\cap T'I'$, and $D=NI'\cap T'F'$ are on $(ABC)$, $NR$ and $T'F$ intersect on $(ABC)$ by the prism lemma.

Let $X'=NK\cap T'F$. Then, $\angle MX'F=\angle MX'T'=90-\angle T'X'N=90-\angle PMI = \angle MIP=\angle MIF$. So, $MIFX'$ is cyclic, so $X'=X$.
Z K Y
The post below has been deleted. Click to close.
This post has been deleted. Click here to see post.
InterLoop
250 posts
#22 • 1 Y
Y by Om245
here is a collection of results obtained using solely Pascal's theorem which i couldnt be bothered to sort out (in some order they give the solution :P)
surely a huge fakesolve idk
in total Pascal's is used $\boxed{6}$ times
solution
Attachments:
This post has been edited 2 times. Last edited by InterLoop, Oct 9, 2024, 2:39 AM
Z K Y
The post below has been deleted. Click to close.
This post has been deleted. Click here to see post.
bin_sherlo
672 posts
#23 • 1 Y
Y by ihategeo_1969
Let $T$ be $A-$mixtilinear touch point with $(ABC)$. Since $AT$ and $AE$ are isogonals, $A,F,T$ are collinear. Also note that $M,I,T$ are collinear. $ND\cap IF=J,NM\cap IJ=P,AE\cap (ABC)=U,TJ\cap (ABC)=W,GT\cap IF=V$. We have $TU\parallel BC$. Observe that $I,J$ are symetric to $MN$. $A,P,U$ are collinear.
Claim: $VFI\sim VIJ\iff VI^2=VF.VJ$.
Proof: Let's show that $(\overline{TV},\overline{TU}),(\overline{TI},\overline{TI}),(\overline{TA},\overline{TJ})$ is an involution. Project onto $(ABC)$. Proving $(G,U),(M,M),(A,W)$ is equavilent to showing the concurrency of $GU,AW$ and the tangent to $(ABC)$ at $M$. Since $\measuredangle MAN=90=\measuredangle NPI$, $(M,I,N,IP\cap TN)$ is an orthogonal system thus, $MA,IP,TN$ are concurrent.
\[(A,T;G,M)\overset{N}{=} (I,NT\cap IJ;F,P)\overset{A}{=} (N,M;T,U)=-1\]Hence $MM,GG,AT$ are concurrent. Apply pascal on $GGUTAW$ to see that $GG\cap TA,GU\cap AW,BC_{\infty}$ are collinear. Since the line passing through both $GG\cap AT$ and $BC_{\infty}$ is $MM$, we get that $GU,AW,MM$ are concurrent.$\square$
We have
\[\frac{NJ}{ND}.\frac{KD}{KI}.\frac{VI}{VJ}=\frac{NJ}{ND}.\frac{AD}{IF}.\frac{VF}{VI}=\frac{NJ}{IF}.\frac{IJ}{NJ}.\frac{VF}{IV}=\frac{IJ}{IV}.\frac{VF}{IF}=1\]Thus, $N,K,V$ are collinear. Let $\overline{NKV}$ intersect $(ABC)$ at $S$. Applying pascal on $MSNAGT$ to get $MS\cap AG,V,I$ are collinear. Hence $L,S,M$ are collinear which is equavilent to $ML\perp NK$ as desired.$\blacksquare$
Z K Y
The post below has been deleted. Click to close.
This post has been deleted. Click here to see post.
bjump
996 posts
#24 • 2 Y
Y by Amkan2022, MathLuis
Solved while talking to MathLuis, kind of a mess of a solution :juggle:

Let $AE \cap (ABC)=J$, $O= AF \cap (ABC)$, Let, $H= ML \cap (ABC)$.
Claim: $(AGFI)$ Cyclic
Proof:
$$\measuredangle AGF = \measuredangle AGN = \measuredangle ABC + \measuredangle BAN = \measuredangle (AB, IF) + \measuredangle BAI = \measuredangle AIF. \square$$
Also, since $LM \cdot LH = LG \cdot LA = LF \cdot LI$ implies $(HFIM)$ cyclic.

Pascal on $(AGNMHJ)$ gives us: $L - P - HJ \cap NG$. So $J-F-H$. Pascal on $AJHDOA$ gives us gives us $I-P-F-S$ where $P$ is $MN \cap IF \cap AE$ (well known incenter config). S is where the tangent line from $A$ on $(ABC)$ intersects $HD$.

Pascal $(OONAAM)$ gives the intersections to the tangents of $(ABC)$ at $O$, and $A$ , $ON \cap AM$, and $I$ are collinear. Observe that by right angles $(AMPI)$ cyclic, and $(IPON)$ cyclic. So by radical axis $I-P- AA \cap OO$. So $S = AA \cap OO$.

Now we have $-1=(NM; AD) \stackrel{O} = (KD \cap ON,I; K, D) $, and $-1 = (OH; AD)$ So by prism lemma $N-H-K$ collinear. Finishing.
Attachments:
This post has been edited 1 time. Last edited by bjump, Dec 21, 2024, 2:49 AM
Z K Y
The post below has been deleted. Click to close.
This post has been deleted. Click here to see post.
cursed_tangent1434
565 posts
#25 • 2 Y
Y by ihategeo_1969, MathLuis
Probably the last solution to this problem for this year! Used a projective transformation just to show that I can, I have a funny feeling like you dont need to. Didn't take me long, and it wasn't as hard as medium China TST geometry problems are made out to be.

Let $\Omega$ denote the circumcircle of $\triangle ABC$. Notice how the problem is equivalent to showing that $\overline{ML}$ and $\overline{KN}$ concur on $\Omega$. Let $T$ denote the $A-$mixtilinear intouch point and let $\ell$ denote the line through $I$ parallel to $\overline{BC}$. The angle condition simply tells that $F$ is the intersection of $\overline{AT}$ and $\ell$. Let $D'$ be the second intersection of line $\overline{DI}$ with $\Omega$ and $T'$ the reflection of $T$ across the perpendicular bisector of segment $BC$. Note that since arcs $TN$ and $T'N$ are equal by symmetry, it follows that points $A$ , $E$ and $T'$ are collinear. We start off by making the following observation.

Claim : Points $G$ , $I$ and $T'$ are collinear.

Proof : Note that by Pascal's Theorem on hexagons $AAMTTN$ and $ANNTMM$, it follows that points $AN \cap MT = I$ , $AA \cap TT$ , $AM \cap NT$ and $NN \cap MM = \infty$ are collinear. Thus, the tangents to $\Omega$ at $A$ and $T$ intersect on $\ell$. Further note that by Pascal's Theorem on $AATMGN$, points $AT \cap GN = F$ , $MT \cap AN = I$ and $MG \cap AA$ are collinear. Thus, $\overline{MG}$ passes through the intersection of the tangents to $\Omega$ through $A$ and $T$ implying that $AGTM$ is harmonic. Thus,
\[-1=(AT;GM)\overset{I}{=}(N,M;,T,GI \cap \Omega)\]which implies that points $G$ , $I$ and $T$ are indeed collinear as claimed.

Take a homography sending $ATNM$ to a rectangle while preserving its circumcircle.

[asy]
import geometry;
size(10cm);
defaultpen(fontsize(9pt));

pair foot(pair P, pair A, pair B) {
return foot(triangle(A,B,P).VC);
}

pen pri; pri=RGB(24, 105, 174);
pen sec; sec=RGB(217, 165, 179);
pen tri; tri=RGB(126, 123, 235);
pen fil=invisible;
pen sfil=invisible;
pen tfil=invisible;

pair A=dir(110);
pair T = dir(250);
pair N = dir(290);
pair M= dir(70);
pair I = intersectionpoint(line(A,N),line(T,M));
pair F = midpoint(A--T);
pair G = intersectionpoints(circle(A,T,N),line(N,F))[1];
pair L = intersectionpoint(line(A,G),line(F,I));
pair D = intersectionpoints(line(foot(G,M,N),G),circle(A,M,N))[1];
pair D_1= intersectionpoints(line(D,I),circle(A,M,N))[0];
pair K = intersectionpoint(line(A,T),line(D,D_1));
pair X = intersectionpoint(line(L,M),line(K,N));

filldraw((path)(A--T--N--M--cycle), white+0.1*pri, pri);
filldraw(circumcircle(A,N,M), tfil, tri);
draw(A--L,sec);
draw(L--I,sec);
draw(G--N,pri);
draw(D_1--D,pri);
draw(D_1--M,pri+dashed);
draw(L--M,dashed+tri);
draw(X--N,dashed+tri);

dot("$A$",A,dir(A));
dot("$T$",T,dir(T));
dot("$N$",N,dir(N));
dot("$M$",M,dir(M));
dot("$I$",I,dir(90));
dot("$F$",F,dir(170));
dot("$G$",G,dir(G));
dot("$L$",L,dir(L));
dot("$D$",D,dir(D));
dot("$D'$",D_1,dir(D_1));
dot("$K$",K,dir(340));
[/asy]

By Pascal's Theorem on hexagon $AMMTNN$, points $AM \cap NT$ , $MM \cap NN = \infty$ and $AN \cap MT = I$ are collinear. Thus, lines $\overline{AM}$ , $\overline{NT}$ and $\ell$ concur. Thus, $\ell$ maps to the line through $I$ parallel to sides $AM$ and $NT$.

We start off by making the following observation, which is a lot easier to see after the projective transformation.

Claim : Lines $\overline{DG}$ , $\overline{AT}$ and $\overline{LM}$ concur.

Proof : Since $\ell$ is parallel to the sides of the rectangle with center $I$, $F$ is the midpoint of $AT$. Let $F'$ denote the reflection of $F$ across $I$. Thus, $F'$ is also the midpoint of $MN$. Thus, $FN \parallel AF'$. Let $S = LM \cap AT$. Then,
\[\frac{GL}{AG}=\frac{FL}{F'F}= \frac{FL}{AM} = \frac{FS}{AS}\]which implies that $GS \parallel \ell$. Further since the diagonals of quadrilateral $GD'T'D$ intersect at the center $I$, it must be a rectangle and thus, $GD \perp DT' \parallel MN$ which implies that $GD \parallel \ell$ as well. This implies that points $G$ , $S$ and $D$ are collinear, which proves the claim.

Claim : Points $M$ , $F$ and $D'$ are collinear.

Proof : Since in the original diagram, $D$ is the reflection of $A$ across $\overline{MN}$ , quadrilateral $AMDN$ is harmonic. Thus,

\[-1=(MN;AD)\overset{I}{=}(TA;ND')\overset{M}{=}(T,A;\infty , AT \cap MD')\]which implies that $\overline{MD'}$ bisects segment $AT$, which implies the claim.

Now, let $X = \overline{KN} \cap \Omega$. By Pascal's Theorem on hexagon $XNGDD'M$, it follows that points $XN \cap DD' = K$ , $NG \cap D'M = F$ and $GD \cap MX$ are collinear. Thus, points $X$ , $S$ and $M$ are collinear. We already proved that points $L$ , $S$ and $M$ are collinear, which implies that $X$ also lies on line $\overline{LM}$, which implies that indeed lines $\overline{NK}$ and $\overline{LM}$ intersect on $\Omega$, as desired.
This post has been edited 1 time. Last edited by cursed_tangent1434, Dec 31, 2024, 12:37 AM
Reason: left out a part of the proof oops
Z K Y
The post below has been deleted. Click to close.
This post has been deleted. Click here to see post.
HamstPan38825
8857 posts
#26 • 1 Y
Y by ehuseyinyigit
Average Chinese TST problem: write a good geometry problem, then add a bunch of lines and points to hide the original problem.

We will use a ridiculous number of phantom points. Let $Q$ be the tangency point of $\Omega$ with the $A$-mixtilinear incircle, $P = \overline{DI} \cap \Omega$, $R = \overline{NQ} \cap \overline{AP}$, and $F' = \overline{AK} \cap \overline{RI}$. Then, in triangle $ANP$, the cevians $\overline{NR}$, $\overline{PI}$, $\overline{AF'}$ concur at $K$.

Claim: $\overline{AA}$, $\overline{NP}$, $\overline{RI}$ meet at a point $U$.

Proof: Let $U_1 = \overline{NP} \cap \overline{RI}$; then $(U_1F'; RI) = -1$ by definition. On the other hand, setting $U_2 = \overline{RI} \cap \overline{AA}$ and using the fact that $\overline{QI}$ passes through $M$ (see #3 here), \[-1 = (AD; NM) \stackrel I= (NP; AQ) \stackrel A= (IR; F'U_2).\]So $U_1=U_2$, as needed. $\blacksquare$

Claim: $\overline{UI} \parallel \overline{BC}$.

Proof: We use another phantom point. Let $\ell$ be the line through $I$ parallel to $\overline{BC}$, and let $U_3 = \ell \cap \overline{AA}$. Now, $\measuredangle U_3IP = \measuredangle ADP = \measuredangle ANP$ (or Reim's theorem :P) so $\overline{U_3I}$ is tangent to $(IPN)$. Next, let $\omega$ be the circle tangent to $\overline{U_3A}$ at $A$ and tangent to $\overline{U_3I}$ at $I$. As $\overline{U_3I}\parallel \overline{NN}$, a homothety at $A$ takes $\omega$ to $\Omega$. So $\overline{AA}$ is the radical axis of $\omega$ and $\Omega$, i.e. $U_3$ is the radical center of $(IPN)$, $\omega$, and $\Omega$. It follows that $U_3$ lies on $\overline{PN}$, so $U_3 = U$. $\blacksquare$

The previous claim and definition of $Q$ together imply $F' = F$. Now let $T = \overline{NR} \cap \Omega$.

Claim: $F$ lies on $\overline{PM}$.

Proof: Let $M' = \overline{PF} \cap \Omega$. Then $-1 = (UF; RI) \stackrel P= (NM';AD)$ so $AM'=M'D$, i.e. $M' = M$. $\blacksquare$

Now by Pascal on $TNGAPM$, $R = \overline{TN} \cap \overline{AP}$, $F = \overline{NG} \cap \overline{PM}$, $L' = \overline{GA} \cap \overline{TM}$ are collinear along the line $\ell$. But $L$ also lies on $\overline{AG}$ and $\ell$, hence $L = L'$. Then $\overline{LM}$ passes through $T$, where it intersects $\overline{NK}$ on $\Omega$, as needed.
This post has been edited 2 times. Last edited by HamstPan38825, Jan 10, 2025, 12:28 AM
Z K Y
N Quick Reply
G
H
=
a